Class #5
A coupon bond that pays interest semiannually has a par value of $1,000, matures in 7 years, and has a yield to maturity of 7.5 percent. If the annual coupon rate is 9 percent, what is the approximate value of the bond today? A) $856 B) $1,000 C) $1,081 D) $1,083
Hide question 5 feedback The correct answer is (C). P/YR =2N =14I =7.5PMT =$45FV =$1,000PV =$1,080.55
A fixed-income portfolio consists of Bond A, Bond B, and Bond C. The bonds have durations of 5, 8, and 10, respectively. If 40% is invested in Bond A and 30% invested in each of the other two bonds, what is the duration for the portfolio? A) 7.4 B) 7.7 C) 8.0 D) 10.0
The correct answer is (A). (0.4 × 5) + (0.3 × 8) + (0.3 × 10) = 2 + 2.4 + 3 = 7.4
What is a bond issued and supported only by the general credit standing of the issuing corporation called? A) Debenture B) Indebenture C) Term bond D) Serial bond
The correct answer is (A). A debenture is insured at the general credit of the issuer.
A normal yield curve is one in which: A) Interest rates gradually increase as bond maturities increase. B) Interest rates gradually decrease as bond maturities increase. C) Interest rates are unaffected by bond maturities. D) Interest rates are generally flat for bond maturities of up to about 10 years, but then rise rapidly for bond maturities of 10 to 30 years.
The correct answer is (A). A sign of a normal yield curve is that interest rates gradually increase as bond maturities increase.
An order to buy or sell a certain quantity of a security at a specific or better price, but only after a specified price has been reached, is called a A) stop-limit order. B) stop-loss order. C) stop order. D) limit order.
The correct answer is (A). A stop-limit order is an order to buy or sell a certain quantity at a specific or better price once a stop price has been reached.
Eric wants to purchase a bond portfolio to fund his daughter's education. If she starts college in 8 years, which of the following is most important to Eric? A) That the bond portfolio has a duration of 8 years. B) That the bond portfolio has a maturity of 8 years. C) That the bond portfolio predominantly consists of coupon bonds. D) That the bond portfolio predominantly consists of callable bonds.
The correct answer is (A). Because Eric has a definite time horizon, he should immunize his portfolio by setting his portfolio's duration equal to his time horizon (in this case, 8 years). Answer (B) is incorrect because the maturity of the bonds in the portfolio is far less important than their duration. Answer (C) is incorrect because Eric is unlikely to care whether his bonds pay coupons; he is interested in funding a future goal, not in receiving income. Answer (D) is incorrect because Eric would likely prefer not to have his bond portfolio called back before his time horizon.
Jayden invested $3,000 in CJD stock, $3,000 in JCD stock, and $3,000 in JFD stock. CJD, JCD, and JFD have expected returns of 6 percent, 7 percent, and 10 percent, respectively. What is the weighted expected return for Jayden's portfolio? A) 7.67% B) 7.00% C) 6.76% D) 4.20%
The correct answer is (A). Because the amount invested in each stock is the same, the weighted average equals the simple average of the returns, or 7.67%.
Blue-chip stocks are most likely A) issued by firms that have well-known brand-name product lines. B) characterized as having high growth potential. C) expected to outperform during economic expansions. D) securities with high levels of systematic risk.
The correct answer is (A). Blue-chip stocks are those that are supported by famous brand names and large corporations. Blue-chip firms are stable, have generated substantial operating cash flow for many years, and are expected to continue being market and industry leaders in the future.
Beth, who lives in New York City, is in the 25 percent federal tax bracket and 6 percent state income tax bracket. Which of the following bonds that she is considering purchasing has the highest after-tax yield? I. A Treasury bond paying 5.4 percentII. A corporate bond paying 5.5 percentIII. A Florida municipal bond paying 4.2 percent A) I only B) II only C) III only D) I and II, which are the same and the highest
The correct answer is (A). Corporate bonds are subject to federal and state income tax. Treasury bonds are subject to federal income tax only. Municipal bonds are not subject to federal income tax but are subject to state income tax if they are not issued by the taxpayer's state of residence.Treasury = 4.050%Corporate = 3.795%Florida Bond = 3.948%
Emily has just retired and wants to purchase a bond portfolio that will provide her with income each year. Which bond portfolio strategy is most appropriate for her? A) The ladder strategy B) The bullet strategy C) The barbell strategy D) The passive strategy
The correct answer is (A). Emily would prefer a ladder strategy in which her bonds mature regularly to provide her with income.
Which of the following is true of over-the-counter markets but not of organized exchanges? A) Trading may occur after regular business hours. B) They allow investors to trade in equities. C) Retail investors may trade with both each other and with institutional investors. D) Trading happens electronically.
The correct answer is (A). OTC markets allow investors to trade in securities after exchanges are closed. All of the other statements describe other OTC markets and exchanges.
Unique aspects of real estate investing as alternative investments most likely include A) the ability to generate regular income. B) low default risk. C) little environmental risk. D) high correlations with debt and equity securities.
The correct answer is (A). One unique aspect of real estate as an alternative investment is the simple fact that leases pay off regularly, typically on a monthly basis. Real estate investing involves significant default and environmental risks.
A commodity that is considered a natural resource that can be mined or extracted and used without much processing is known as a A) primary commodity. B)secondary commodity. C) soft commodity. D) hard commodity.
The correct answer is (A). Primary commodities are those that are considered natural resources that can be mined or extracted and used without much processing needed for consumption. Examples include oil, gold, and wheat. When a commodity must undergo a significant process to be able to be consumed, it is known as a secondary commodity, which includes refined gas, lumber, and steel.
Which of the following stocks would be considered the most defensive? A) ABC has a beta of 0.5. B) DEF has a beta of 1.0. C) GHI has a beta of 1.5. D) JKL has a beta of 100.
The correct answer is (A). The beta of the market is 1.0. A stock with a beta between 0 and 1.0 will be less volatile than the market and, therefore, defensive. A stock with a beta greater than 1.0 will be more volatile than the market.
Which of the following is the best reason to invest in alternative investments? A) To achieve additional risk reduction through the addition of a low correlation asset B) To improve current income and achieve a higher rate of investment return C) To increase the portfolio return through the addition of a liquid asset to the portfolio D) To increase the diversification of the portfolio with a low-cost, moderate-risk asset
The correct answer is (A). The major advantage of alternative investments is their historically low correlation with stocks and bonds. The primary purpose of alternative investments is to reduce risk through the addition of a low correlation asset class. Option (B) is incorrect because alternative investments often do not produce any current income and may or may not produce high returns. Option (C) is incorrect since most alternative investments are illiquid and may or may not produce high returns. Option (D) is incorrect because the typical alternative investment has high fees and is only suitable for sophisticated investors with high risk tolerance.
Ian Chesterton, chartered financial analyst®, uses fundamental analysis to evaluate equity securities. Chesterton has been able to use inflation data, historical dividend information, and price-to-book ratios to consistently outperform a broad equity index. Chesterton would most likely identify markets as being A) weak-form efficient. B) strong-form efficient. C) semistrong-form efficient. D) all of the above
The correct answer is (A). The use of fundamental analysis to generate abnormal returns violates the semistrong and strong forms of the efficient market hypothesis. Using publicly available information such as dividends and inflation would not result in outperformance in a semistrong-form or strong-form market. Therefore, the market can be only a weak-form level of efficiency.
How does the constant-weighting strategy differ from other asset allocation strategies? A) The constant-weighting strategy requires a fund manager to regularly return to the strategic asset allocation. B) Only under the constant-weighting strategy is the asset allocation unable to change over time. C) Under the constant-weighting strategy, fund managers are constantly looking for ways to change the asset weights to beat the market. D) The constant-weighting strategy is especially focused on fixed-income securities.
The correct answer is (A). Under the constant-weighting strategy, the fund manager will set up a pre-determined rule for when to return to the strategic asset allocation. Generally, the rule is that the fund cannot have an asset allocation too different from the strategic asset allocation. For example, a manager who wants a 70/30 stock/bond allocation will rebalance if the portfolio allocation rises to 75/25 or falls to 65/35. Another common rule is to rebalance back to the strategic asset allocation on a fixed schedule such as at the beginning of every quarter.
If a bond is trading at a premium, which of the following will be highest? A) The coupon rate B) The current yield C) The yield to maturity D) All of the above will be equal
The correct answer is (A). When a bond is trading at a premium, its coupon rate will be higher than its current yield which, in turn, will be higher than its yield to maturity.
Dinah's portfolio consists of a 50 percent equity index fund and a 50 percent fixed-income index fund. More details are shown below: Fund Shares Net Asset Value Total Value Equity Index 2,000 $25.00 $50,000 Fixed-income Index 1,250 $40.00 $50,000 A year later, the price of the equity fund changes to $20.00 and the price of the fixed-income fund changes to $50.00. All of the following statements correctly describe how these price changes affected the portfolio EXCEPT: A) The portfolio likely has a riskier asset allocation. B) The expected return of the portfolio has likely decreased. C) The portfolio's style is drifting away from balance and more toward fixed-income securities. D) To rebalance her portfolio, Dinah must sell shares of the fixed-income index and buy shares of the equity index.
The correct answer is (A).The allocation after the changes in prices is as follows: Fund Shares Net Asset Value Total Value New Allocation Equity Index 2,000 $20.00 $40,000 39.02% Fixed-Income Index 1,250 $50.00 $62,500 60.98% Dinah's portfolio is now weighted more toward fixed-income securities. This means her portfolio is likely less risky than it was before and is also likely to have a lower return. To rebalance back to a 50/50 allocation, Dinah must sell off shares of the fixed-income index and buy shares of the equity index.
The John Fund earns 10 percent during the year while the risk-free rate is 3 percent. The John Fund has a beta of 1.10 and a standard deviation of 15 percent. Meanwhile, the market portfolio returned 8 percent with a standard deviation of 10 percent. The Sharpe ratio is closest to A) 0.1333. B) 0.4667. C) 1.8182. D) 6.3636.
The correct answer is (B). Sharpe = (Portfolio Return - Risk-Free Rate) ÷ Standard Deviation Sharpe = (10 − 3) ÷ 15 = 0.4667
A mutual fund with a beta of 0.90 and a standard deviation of 11% earns an 8% return during a year in which the risk-free rate is 3 percent. Meanwhile, the market portfolio returned 9% with a standard deviation of 10%. The mutual fund's Sharpe ratio is closest to A) -0.4000. B) 0.4545. C) 0.7273. D) 5.5556.
The correct answer is (B). Sharpe = (Portfolio Return - Risk-Free Rate) ÷ Standard Deviation Sharpe = (8 - 3) ÷ 11 = 0.4545
A fund generated a return of 10 percent. Meanwhile, the market returned 11 percent and the risk-free return was 3 percent. If the fund's beta is 0.9, then Jensen's alpha for the fund is closest to which of the following? A) -2.6 B) -0.2 C) +6.2 D) +7.8
The correct answer is (B). a = rp - [rf + (rm - rf) x β] a = 10 - [3 + (11 - 3) x 0.9] a = 10 - [3 + (8 x 0.9)] a = 10 - [3 + 7.2] a = 10 - 10.2 = -0.2
The return on a certain derivative security is usually small and positive. But, in rare cases, its returns is very large and negative. How would you describe the distribution of this return? A) Positively skewed B) Negatively skewed C) Platykurtic D) Leptokurtic
The correct answer is (B). A distribution with a small number of extremely low observatives is said to have a negative skew.
Which type of underwriting arrangement is the riskiest to the underwriter? A) A best-efforts agreement B) A firm commitment C) A private placement D) A standby underwriting
The correct answer is (B). A firm commitment puts all the risk on the underwriter.
Compared to money-market instruments, an advantage of equities is their relatively high A) liquidity. B) expected return. C) risk level. D) availability.
The correct answer is (B). A major advantage of equities over other asset classes is their relatively high expected return. Both equities and money-market instruments are highly liquid and available in the secondary market. That equities are riskier than money-market instruments is a disadvantage, not an advantage.
One investor uses fundamental analysis to form a portfolio of equity securities and outperformed his benchmark for almost a decade. Another investor forms a portfolio using technical analysis but has never beat her benchmark. The market under these conditions is most likely A) inefficient. B) weak-form efficient. C) semistrong-form efficient. D) strong-form efficient.
The correct answer is (B). A semistrong-form market is one in which fundamental analysis would not lead to superior returns. Thus, a market that provides persistent out-performance opportunities must be only weak-form efficient.
Assume that the 3-month return for Hartfield stock is 3 percent. What is the annualized return for Hartfield? A) 13.20% B) 12.55% C) 12.00% D) 11.85%
The correct answer is (B). Annualized return = (1.03)⁴ − 1 = 12.55%
Nate invested in the Not-So-Good mutual fund 5 years ago. His returns were -5 percent, -8 percent, -5 percent, -6 percent, and -8 percent, respectively. What is the arithmetic average return over the 5 years? A) -8% B) -6.4% C) 0% D) 6.4%
The correct answer is (B). Arithmetic average = (-5% - 8% - 5% - 6% - 8%) / 5 = -6.4%.
Harriet owns many shares of a company. She wants to sell those shares because she needs cash to pay for repairs to her home. Which of the following individuals is most well-suited for helping her to liquidate her shares? A) A broker-dealer who works in the primary market B) A broker-dealer who works in the secondary market C) An underwriter who utilizes the best-efforts method D) An underwriter who utilizes the firm commitment method
The correct answer is (B). Harriet can liquidate her shares in the secondary market. To help facilitate those trades, she can work with a broker-dealer. Answer (A) is incorrect because Harriet cannot sell her shares in the primary market; that market is for companies selling shares directly to investors. Answers (C) and (D) are incorrect because underwriters work with companies to sell their shares on the primary market.
The Jensen Hybrid Fund would most likely be chosen as an investment vehicle by A) investors seeking high returns on equity investments. B) investors seeking to plan for retirement. C) investors seeking growth opportunities. D) investors seeking low tax liabilities.
The correct answer is (B). Hybrid funds are very popular as target retirement date planning tools. Answers A and C are incorrect because a hybrid fund is not likely to generate high returns or considerable growth. Answer D is incorrect because hybrid funds include bonds, the income of which is taxed at relatively high ordinary income rates.
John Enterprises (JE) has an average return of 3 percent with a standard deviation of 6 percent. Assuming the returns are normally distributed, what is the probability that JE will have a return greater than 9 percent? A) 2.5% B) 16% C) 34% D) 66%
The correct answer is (B). Nine percent is one standard deviation to the right of the mean. Because its returns are normal, 68 percent will be between +1 and -1 standard deviations from the mean. Meanwhile, 16 percent of returns will be even greater than +1 standard deviations from the mean while another 16 percent of returns will be even less than -1 standard deviations from the mean.
All of the following statements concerning risk are correct EXCEPT: A) Country risk, or political risk, is the variability in a security's returns resulting from the instability of a country's economy or government. B) Financial risk is associated with the use of equity as part of a company's capital structure. C) Market risk is the variability in a security's returns resulting from fluctuations in the overall market. D) Business risk is the risk associated with the industry or environment in which a business operates.
The correct answer is (B). Options (A), (C), and (D) are true statements. Option (B) is false because financial risk deals with debt as part of a capital structure, not equity.
Which of the following dates related to dividends has been most affected by recent changes in technology and brokerage policies? A) The declaration date B) The ex-dividend date C) The record date D) The payment date
The correct answer is (B). Over the past few decades, the ex-dividend date has moved closer-and-closer to the record date. Changes in technology and in brokerage policies have reduced the time necessary to process orders, allowing investors to own securities more quickly after purchasing them.
Consider the chart with the seven portfolios and the efficient frontier. Which of the following portfolios is the most efficient in terms of risk and return? A) Portfolio A B) Portfolio D C) Portfolio E D) Portfolio F
The correct answer is (B). Portfolios B, C, and D are all efficient.Portfolios A, E, and F are all inefficient as there are other attainable portfolios with a higher return for the given level of risk.Portfolio G is unattainable.
All of the following are common roles for private equity firms EXCEPT A) raising capital from wealthy individuals and institutions. B) acting as limited partners in the business relationship. C) identifying potentially undervalued businesses. D) offering strategic advice on the direction of the business.
The correct answer is (B). Private equity firms act as the general partner, and the investors act as the limited partners.
Which of the following mutual fund types will most likely have the highest total risk level? A) A large-cap index fund B) A small-cap growth fund C) A large-cap international fund D) A mid-cap domestic equity fund
The correct answer is (B). Small-cap growth funds invest primarily in smaller firms that generally pay no dividends and reinvest all of their cash flows back into the firm to finance growth opportunities. These types of funds have more risk than index funds and large-cap funds.
Glen is considering two portfolios: 1) Portfolio A with a return of 14 percent and a standard deviation of 14 percent and 2) Portfolio B with a return of 4 percent and a standard deviation of 7 percent. Assuming the correlation between A and B is 0.0 and Glen invests 70 percent in A and 30 percent in B, what range of returns should this portfolio produce 95 percent of the time? A) Between 1% and 21% B) Between -9% and 31% C) Between -1% and 21% D) Between -19% and 41%
The correct answer is (B). Standard deviation: =SQRT[(0.7² x 14² ) + (0.3² x 7² ) + 0]=SQRT[(0.49 x 196) + (.09 x 49)]=SQRT[96.04 + 4.41]=SQRT[100.45]=10.02% or about 10% Expected return = (0.7 × 14%) + (0.3 × 4%) = 11%As noted in Chapter 2, the 95 percent confidence interval equals two standard deviations from the mean. Therefore the range -9 percent to 31 percent is the correct answer.
Rich, Inc.'s, stock pays a $5 dividend quarterly. Its current earnings per share is $25. If the stock is currently trading at $400, what is the dividend yield percentage? A) 1.25% B) 5% C) 20% D) 25%
The correct answer is (B). The dividend yield percentage is equal to the total annual dividends per share divided by the stock price.($5 × 4) / $400 = 5%
Warren Buffet famously said, "You only find out who is swimming naked when the tide goes out." The quote, in part, means that when the equity market is increasing, it hides many of the weaknesses of businesses. In this quote, what type of risk does the rising and falling tide represent? A) Interest rate risk B) Market risk C) Business risk D) Financial risk
The correct answer is (B). The increase or decrease in the market has an impact on most companies. This risk is referred to as market risk.
Which of the following is not part of the U.S. Securities and Exchange Commission's mission? A) Protect investors. B) Insure against large losses for issuers in the primary market. C) Maintain fair, orderly, and efficient markets. D) Facilitate capital formation.
The correct answer is (B). The mission of the U.S. Securities and Exchange Commission is to protect investors; maintain fair, orderly, and efficient markets; and facilitate capital formation.
What is a disadvantage of exchange-traded funds (ETFs)? A) Like stocks and bonds, they can be bought and sold at any time during market hours. B) They can be subject to brokerage commissions. C) They can transfer securities out to redeeming shareholders. D) They represent a basket of stocks and bonds.
The correct answer is (B). The other choices are advantages.
Assume Marcie adds to her portfolio Security Y, which is less than perfectly positively correlated with the portfolio. Security Y has the same standard deviation as the portfolio. What will happen to the standard deviation of the portfolio after she adds Security Y? A) It will remain the same. B) It will decrease. C) It will increase. D) It may increase or decrease, depending on the weightings of the portfolio holdings.
The correct answer is (B). The portfolio standard deviation should decrease because the added security is not perfectly correlated with the portfolio. For example: The portfolio standard deviation is 20 percent, and Security Y's standard deviation is 20 percent. The correlation between the portfolio and Security Y equals 0.95. With a weight of 95 percent for the portfolio and 5 percent for Security Y, the combined standard deviation equals 19.95 percent.
The primary difference between unit investment trusts and mutual funds is that A) only mutual funds pool investments. B) unit investment trusts are far less liquid. C) unit investment trusts have higher ongoing management fees. D) unit investment trusts are not permitted to invest in equities.
The correct answer is (B). The secondary market for unit investment trusts is fairly limited, making them less liquid than mutual funds. Answer (A) is incorrect because both mutual funds and unit investment trusts pool investments. Answer (C) is incorrect because unit investment trusts tend to have low ongoing management fees. Answer (D) is incorrect because both mutual funds and unit investment trusts may invest in equities.
Treasury bills are discount securities most likely because they A) are generally considered to be default risk-free. B) make no explicit interest payments. C) have yields lower than their coupon rates. D) have short maturities.
The correct answer is (B). Treasury bills promise to pay the face value of the bill at maturity, which is typically less than one year. Since no coupon payments are made, the price of the bill almost certainly will sell for less than its face value.
A city funds the construction of a new toll bridge by issuing bonds. Because the bridge is expected to be profitable, the city most likely funded the bridge with which type of bond? A) A general obligation municipal bond. B) A revenue municipal bond. C) A private activity municipal bond. D) A corporate bond.
The correct answer is (B). When a city needs to fund a project that is expected to generate a profit, is it likely to issue revenue municipal bonds that pay based on the revenue generated by the project.
Kevin invested $1.5 million in Fish N' Chips, trading at a market value of $35 per share. A year later, the market value climbed to $40 per share and Kevin invested another $500,000. And, a year after that, the market value dropped to $38 per share. What was Kevin's time-weighted return? A) 2.92% B) 4.20% C) 5.84% D) 8.57%
The correct answer is (B).Time-weighted return can be solved using the CFj button on your calculator: CF0 = <35> CF1 = 0 CF2 = 38 SHIFT IRR/YR Your calculator will then show 4.20%.
The expected market return is 12 percent, and the risk-free rate of return is 3 percent. Using the CAPM formula, what is the expected return for a portfolio that has a standard deviation of 18 percent and a beta of 1.3? A) 12.0% B) 13.8% C) 14.7% D) 15.6%
The correct answer is (C). CAPM = Rf + β(Rm - Rf) CAPM = 3% + 1.3 × (12% − 3%) = 14.7%
Darrel buys Hollandaise, Inc., stock for $112 using a margin account with a 50 percent initial margin and a 35 percent maintenance margin. Assuming the price of the stock drops to $56, how much would Darrel need to pay to restore the equity in his account to the maintenance margin? A) $0 B) $14.80 C) $19.60 D) $22.40
The correct answer is (C). Required equity: $56 × 35% = $19.60Current equity: $56 stock price − $56 loan = $0Margin call: $19.60 - $0 = $19.60
The Miracle Whip Fund (MWF) generated a return of 14.2 percent. Meanwhile, the market returned 10.75 percent and the risk-free return was 3.2 percent. If MWF's beta is 1.3, then Jensen's alpha for the fund is closest to which of the following? A) -2.0 B) +1.96 C) +1.2 D) +5.2
The correct answer is (C). ap = rp − [rf + (rm − rf) × βp] a = 14.2 − [3.2 + (10.75 - 3.2) × 1.3] a = 14.2 − [3.2 + (7.55 × 1.3)] a = 14.2 − [3.2 + 9.815] a = 14.2 − 13.015 = 1.2
Peaches, Inc., expects to generate $50 million in operating cash flows during the next year. It estimates its long-term dividend growth rate to be 3 percent, and it has 50 million shares outstanding. What is the intrinsic value of Peaches, Inc., if your required rate of return is 10 percent? A) $5.88 B) $10.00 C) $14.29 D) $33.33
The correct answer is (C). Intrinsic value = Total market value ÷ Outstanding Shares. Total market value = Operating Cash Flow ÷ (Required Rate of Return - Dividend Growth Rate) Total market value = $50,000,000 ÷ (0.10 − 0.03) = $50,000,000 ÷ 0.07 = $714,285,714 Intrinsic value = $714,285,714 ÷ 50,000,000 = $14.29
The Yoko Fund earns 11.2 percent during the year while the risk-free rate is 3.1 percent. The Yoko Fund has a beta of 1.20 and a standard deviation of 17.5 percent. The Treynor ratio is closest to A) 0.463. B) 0.640. C) 6.750. D) 9.333.
The correct answer is (C). Treynor = (Portfolio Return - Risk-Free Rate) ÷ Beta Treynor = (11.2 − 3.1) ÷ 1.20 = 6.750
Daniela is considering purchasing a 3-year bond that is selling for $1,000. What is the current yield for this bond if it has a 4 percent coupon paid semiannually? A) 2% B) 3% C) 4% D) None of the above
The correct answer is (C). $40 ÷ $1000 = 4%
Which of the following is included in the ultimate price an investor pays for a coupon bond? I. The bond's underlying or 'flat' price.II. Any accrued interest. A) I only B) II only C) Both I and II D) Neither I nor II
The correct answer is (C). A bond's price includes its flat price as well as any accrued interest.
All of the following statements regarding mortgage-backed securities (MBSes) and fixed-rate bonds are correct EXCEPT: A) MBSes pay coupons monthly, while fixed-rate bonds pay coupons semiannually. B) MBSes pay principal monthly, while fixed-rate bonds pay principal at maturity. C) MBSes are subject to interest-rate risk, while fixed-rate bonds are not. D) MBSes are subject to prepayment risk, while fixed-rate bonds are not.
The correct answer is (C). Both fixed-rate bonds and MBSes are subject to interest-rate risk.
Which of the following regarding average returns is (are) correct? I. The geometric mean is equivalent to the time-weighted return.II. The arithmetic mean is the average return for a series of returns and always will be greater than or equal to the geometric mean. A) I only B) II only C) Both I and II D) Neither I nor II
The correct answer is (C). Both statements are correct.
Which of the following statements is (are) correct in regard to the functions of a broker-dealer? I. To act as intermediaries that help clients buy and sell securitiesII. To hold an inventory of securities they believe are undervalued in order to sell them later for a profit A) I only B) II only C) Both I and II D) Neither I nor II
The correct answer is (C). Broker-dealers serve two functions: To act as an intermediary in helping clients buy and sell securities and to generate a profit on the trading of undervalued securities. In their dealer function, broker-dealers can hold inventory when they believe the shares are undervalued with the intent of selling them at a later date when the price rises.
The expected market return is 12 percent, and the risk-free rate of return is 3 percent. Using the CAPM formula, what is the expected return for a portfolio that has a standard deviation of 18 percent and a beta of 1.3? A) 12.0% B) 13.8% C) 14.7% D) 15.6%
The correct answer is (C). CAPM = Rf + β(Rm - Rf)CAPM = 3% + 1.3 × (12% − 3%) = 14.7%
Which of the following features distinguishes Class C mutual funds from other classes of mutual funds? A) They are all "no load." B) They are hybrid funds that include both equity and debt instruments. C) They charge relatively high 12b-1 fees. D) They are only available to institutional and accredited investors.
The correct answer is (C). Class C mutual funds charge relatively high 12b-1 fees but relatively low front- and back-end loads.
The venture capital stage most associated with the transition from private ownership to a public offering is known as the A) startup stage. B) expansion stage. C) bridge stage. D) market stage.
The correct answer is (C). During the bridge stage, the venture capital firm takes steps to move the company toward an initial public offering.
Jayla is considering purchasing a collateralized mortgage obligation (CMO). All of the following are correct EXCEPT: A) CMOs are divided into tranches, which will permit Jayla to better match the timing of the cash flows to her needs. B) CMOs are subject to interest-rate risk, reinvestment-rate risk, and prepayment risk. C) CMOs are not subject to default risk. D) CMOs can help Jayla gain exposure to the real estate market if she is not a homeowner.
The correct answer is (C). Options (A), (B), and (D) are correct. Option (C) is not correct because CMOs are subject to default risk of the underlying mortgages.
Toby recently graduated from college with a degree in finance. He enjoys stock analysis and is eager to get started with investing in the market. He has approximately $30,000 of student loan debt at an average interest rate of 4.2 percent, and the $300 monthly payment is easily manageable even at the starting salary for a new college graduate. Although Toby does not currently have any savings, he has asked a financial planner to assist him with opening a brokerage account where he can begin buying stocks to save toward his goal of buying a condo in the next 7 to 8 years. Which of the following is true? A) Toby's willingness (tolerance) to take on risk is greater than his ability (capacity) to take on risk, so the planner should encourage him to begin investing toward his goal by purchasing stocks in the brokerage account. B) Toby's ability (capacity) to take on risk is greater than his willingness (tolerance) to take on risk, so the planner should encourage him to begin investing toward his goal by purchasing stocks in the brokerage account. C) Toby's willingness (tolerance) to take on risk is greater than his ability (capacity) to take on risk, so the planner should encourage him to accumulate emergency funds prior to purchasing stocks in a brokerage account to save for the goal. D) Toby's ability (capacity) to take on risk is greater than his willingness (tolerance) to take on risk, so the planner should encourage him to accumulate emergency funds prior to purchasing stocks in the brokerage account to save for the goal.
The correct answer is (C). Risk capacity is a measurement of the amount of risk a client can afford to take on. Risk tolerance is the amount of risk a client is willing to take on. When risk capacity and risk tolerance are not in alignment, the more conservative of the two should determine the planning recommendations. In this case, Toby has the tolerance to take on risk; however, since he has not yet accumulated any emergency funds, he lacks the capacity to absorb losses. Therefore, he should build his emergency funds first and then begin investing for other goals.
Which of the following investor acts would most directly allow them to profit off a decline in a security's price? A) A purchase holder B) A stop-loss order C) A short sale D) A capital loss
The correct answer is (C). Short sales allow investors to profit off the decline in a security's price.
Which of the following statements about Treasury bills is correct? A) T-bills are issued maturities of 4 weeks, 13 weeks, and 26 weeks only. B) T-bills that extend beyond a calendar year are subject to taxation on the income earned both in the year of issuance and in the year of maturity. C) T-bills are sold on a pure discount basis only. D) The stop-yield is the lowest yield determined during a T-bill auction.
The correct answer is (C). T-bills are typically auctioned with maturities of 4, 13, 26, or 52 weeks. T-bills are not subject to original issue discount (OID) rules since their maturities are no greater than one year. Option (C) is correct. The stop-yield is the highest yield, not the lowest yield.
Coyote, Inc., has net earnings of $3 billion this year. It has 500 million shares of common stock outstanding, and it paid $0.25 per share per quarter this year as a dividend. Which of the following is correct? A) The retention ratio equals 8.33%. B) The payout ratio equals 8.33%. C) The payout ratio equals 16.67%. D) The retention ratio equals 16.67%.
The correct answer is (C). The dividend per share equals $1.00. The Earnings Per Share (EPS) equals $6.00, which is found by dividing net earnings by outstanding shares. The payout ratio = dividend per share / EPS.
Evan has a portfolio of mutual funds, A, B, and C. She has 50 percent in A, 40 percent in B, and 10 percent in C. What is the expected return for the portfolio if the relative expected returns for A, B, and C are 10 percent, 8 percent, and 14 percent? A) 11.50% B) 10.67% C) 9.60% D) 9.35%
The correct answer is (C). The weighted average expected return equals (0.5 × 10%) + (0.4 × 8%) + (0.1 × 14%) = 9.6%.
Ollie, who is a financial advisor, manages an equity portfolio for the Green Arrow University endowment fund, which has an 8.1 percent return objective. Ollie makes a strategic allocation recommendation that produces a return of 7.8 percent in an economy that has experienced a 2.7 percent rate of inflation. Ollie also creates his own benchmark for the fund, which includes multiple indexes that have similar risk profiles of the securities in the fund. The benchmark return during the period is 7.5 percent. Is the endowment fund satisfied with Ollie's performance? A) Yes, because the fund outperformed inflation. B) Yes, because the fund outperformed the return objective. C) Yes, because the fund outperformed the benchmark return. D) No, because the fund underperformed the objective.
The correct answer is (C). While the fund did not return more than the return objective, it still performed better than the benchmark return.
Nyssa is considering two portfolios: 1) Portfolio A with a return of 11 percent and a standard deviation of 16 percent and 2) Portfolio B with a return of 6 percent and a standard deviation of 10 percent. Assuming the correlation between A and B is 0.0, which of the following is the most efficient portfolio? A) 10% A / 90% B B) 20% A / 80% B C) 30% A / 70% B D) All three combinations of A and B are equally efficient.
The correct answer is (C).The expected return is the weighted average return. The standard deviation of each combination is found using the following formula: Option (A) return and standard deviation = 6.5% and 9.14%, respectively. Option (B) return and standard deviation = 7.0% and 8.62%, respectively. Option (C) return and standard deviation = 7.5% and 8.49%, respectively. The most efficient (highest return for given level of risk) is Option (C). Options (A) and (B) are inefficient.
Bennett is considering purchasing an 8-year bond that is selling for $700. What is the YTM for this bond if it has a 6 percent coupon paid semiannually? A) 12.17% B) 12.14% C) 12.05% D) 11.92%
The correct answer is (D). END Mode P/YR 2 PV -$700.00 N 8 × 2 = 16 PMT $30 FV $1,000.00 i 11.92%
Ira bought 10,000 shares of LeLe at $40 per share. Two years later, she sold the stock for $80 per share. LeLe declared and paid a special dividend of $4 per share during the period Ira held the stock. What was Ira's holding period return (HPR)? A) 10% B) 55% C) 100% D) 110%
The correct answer is (D). HPR = (Income received + Change in price) / Beginning Price HPR = [$4 + ($80 - $40)] / $40 HPR = [$4 + $40] / $40 HPR = $44 / $40 HPR = 110%
The Charlie bond is a 3 percent coupon bond with semiannual coupon payments that matures in 30 years. If the yield to maturity (YTM) for this bond is 6 percent, what is the value of the bond? A) $622.28 B) $602.41 C) $587.06 D) $584.87
The correct answer is (D). P/YR 2 FV $1,000.00 N 60 i 6.00% PMT $15.00 PV $584.87
The Paul Fund earns 14 percent during the year while the risk-free rate is 3 percent. The Paul Fund has a beta of 1.10 and a standard deviation of 18 percent. The Treynor ratio is closest to A) 0.61. B) 0.78. C) 1.00 D) 10.00.
The correct answer is (D). Treynor = (Portfolio Return - Risk-Free Rate) ÷ Beta Treynor = (14 − 3) ÷ 1.10 = 10.00
Generally, fixed-income investors are subject to which of the following risks? A) Purchasing power risk B) Interest-rate risk C) Default risk D) All of the above
The correct answer is (D). All fixed-income investors are subject to purchasing power risk and interest-rate risk, neither of which can be diversified away. Most investors are also subject to default risk.
Which of the following is correct regarding the reasons why an investor might consider investing in corporate bonds? A) They have lower default risk than Treasuries. B)They generally pay lower interest than Treasuries. C) They generally offer some inflation protection via the semiannual payment of coupons. D) They offer higher security of principal than equities.
The correct answer is (D). Corporate bonds have higher default risk and higher interest rates than Treasuries. Semiannual coupon payments offer no inflation protection
Firms are least likely to use the primary equity market to raise capital for A) global expansion. B) research and development investments. C) the launch of new product lines. D) a desire to increase its financial leverage.
The correct answer is (D). Firms use the equity markets for a variety of reasons, but always it is to raise capital for global expansion, for research costs, and for the introduction of new product lines. When firms issue new shares of equity, however, their debt-equity ratios and therefore financial leverage will fall.
When must investment advisors register with the SEC? A) All investment advisors must register with the SEC. B) When their assets under management exceeds $1 million. C) When their assets under management exceeds $10 million. D) When their assets under management exceeds $100 million.
The correct answer is (D). Investment advisors must register with the SEC when their assets under management exceeds $100 million.
A type of preferred stock whose payments, when missed, must be paid prior to paying dividends to common stock is A) preferential preferred stock. B) participating preferred stock. C) non-cumulative preferred stock. D) cumulative preferred stock.
The correct answer is (D). Preferred stock (also known as straight preferred stock) has a right to a fixed dividend. Preferred stock may also be cumulative and/or participating. Cumulative preferred stock has the right to receive unpaid dividends that are in arrears before common shareholders receive any dividends. Participating preferred stock is entitled to share in the profits of the company.
The dividend-payout ratio is equal to A) the dividend yield. B) dividends per share divided by the par value of the stock per share. C) dividends per share divided by the current market price per share. D) dividends per share divided by earnings per share.
The correct answer is (D). The dividend-payout ratio is equal to dividends per share divided by earnings per share.
Consider the graph with the security market line (SML). Which of the following statements is correct? A) The betas for Portfolios A, B, and C are all the same as they are all on the SML. B) The return for Portfolio C will be equal to the market premium minus the risk-free rate. C) If the return for the market was 12 percent and the risk-free rate was 3 percent, then A's expected return could not equal 15 percent. D) Portfolio A is the riskiest portfolio.
The correct answer is (D). The x-axis in this graph is risk. Portfolio A is furthest along the x-axis and therefore the riskiest. Option (A) is incorrect as the beta for C is the market beta of 1.0, while the beta for B is less than 1.0 and the beta for A is more than 1.0. Option (B) is incorrect because the return for C will equal the return for the market, which is also the market premium plus the risk-free rate. Option (C) is incorrect as A's return could equal 15 percent if the beta for A were 1.3333.
Which of these market indexes is price-weighted? A) The S&P 500 B) The Russell 2000 index C) The Wilshire 5000 index D) The Dow Jones Industrial Average
The correct answer is (D).The Dow Jones Industrial Average is is price-weighted rather than value-weighted. It is also unlike the other indexes because it has only 30 companies, while the other indexes have at least 500 companies. And, it is the least representative of the U.S. economy.
How do most investors gain exposure to money market securities? A) By purchasing individual securities on the primary market. B) By purchasing individual securities on the secondary market. C) By depositing cash into money market deposit accounts or buying purchasing money market mutual funds. D) None of the above.
The correct is answer is (C). Money market deposit accounts and money market mutual funds allow investors to indirectly own many money market securities that would otherwise be difficult to invest in individually.